¿Es el "número de fotones" de un sistema un invariante de Lorentz?

Me pregunto si la cantidad de fotones de un sistema es un invariante de Lorentz. Google devuelve un artículo que parece indicar que sí es invariante al menos cuando el sistema es una cavidad rectangular de paredes superconductoras.

Sin embargo, me dijeron en la sala de chat de hbar que no es un invariante y que es proporcional al primer término del 4-momento que está relacionado con el hamiltoniano de la "teoría del campo libre".

Hoy hablé con un amigo que estudia algo de GR (todavía no QFT) y no podía creer que este número no fuera invariante de Lorentz.

Así que, en general, me quedo confundido. ¿Es un invariante de Lorentz para algunos sistemas y no para otros? Si es así, ¿cuáles son las condiciones que debe cumplir un sistema para que el número de fotones sea invariable?

Un número contado es un número contado, sin importar en qué sistema de coordenadas lo escribas (no te conviertes en trillizos enviando a tu gemelo a una misión espacial en un cohete realmente rápido). El primer término en los cuatro impulsos sería una energía y, por supuesto, se transforma bajo una transformación de Lorentz. Ese es el efecto Doppler. No creo que esto sea tan trivial para el caso de los fotones térmicos, que para empezar no tienen un número fijo. Solo es significativo el número medio de fotones de un estado térmico.
@CuriousOne Encontré la discusión del chat: chat.stackexchange.com/transcript/71?m=26894546#26894546 , parece que tanto FenderLesPaul como ACuriousMind acordaron que no es un invariante... Me encantaría saber más sobre los detalles.
No me lo tomaría demasiado en serio. No es una pregunta simple, por cierto. El número de fotones definitivamente NO es un invariante en los sistemas de coordenadas acelerados. No creo que sea un invariante en el caso de los fotones térmicos que tienen que estar en equilibrio termodinámico con un baño térmico invariante que no sea de Lorentz. Si lanzas siete átomos en un volumen fijo y haces que emitan siete fotones, esos siete fotones no térmicos siempre permanecerán siete en cualquier sistema de coordenadas, aunque... así que es un sí, pero...
Siete fotones detectados serían lo mismo para cualquier observador; no hay forma de saber acerca de los fotones sin detección.
Consulte core.ac.uk/download/files/425/11921990.pdf al final de la página 9. Las representaciones unitarias del espacio de Hilbert no equivalentes de una QFT corresponden a diferentes observadores para los que el operador numérico puede ser diferente. Esta es esencialmente la falla del teorema de Stone-von Neumann. Vea esta respuesta physics.stackexchange.com/questions/176652/… también
@ user40276 ese documento parece abordar la diferencia entre marcos inerciales y no inerciales. Pero por lo que puedo decir, no contradice la afirmación de que el número de fotones es el mismo en cualquier marco inercial y, por lo tanto, es invariante de Lorentz.
@CuriousOne Eso debería, con una ligera expansión, ser una respuesta.
@JT: Dejaré que un teórico tome este. Para mi gusto, eso es demasiado agitar las manos en una pregunta importante. Estoy seguro de que uno puede dar una respuesta mucho mejor que esa. Si mi intuitivo está de acuerdo con el teórico correcto, tomaré un trago de la casa, pero no quiero afirmar que tengo suficiente experiencia en un área en la que solo puedo improvisar.
@Rococó Sí. Mi afirmación fue que los observadores generales detectarán un número diferente. Por supuesto, pueden estar en marcos totalmente diferentes no relacionados por una transformación de Lorentz o incluso pueden no estar en un marco. Mi punto es más filosófico que teórico. El espacio de Fock es solo una de las representaciones posibles y, por lo tanto, diferentes "interpretaciones" de la misma QFT conducen a un operador numérico diferente.
@Rococo Por ejemplo, si tenemos dos representaciones no equivalentes de un sistema libre junto con la dinámica, a la vez t en un marco dado en una variedad, el operador numérico puede alcanzar diferentes valores en cada sistema, pero ni siquiera son comparables, porque viven en diferentes espacios de Hilbert.
@ user40276 está bien, gracias por la aclaración. Ciertamente es un punto interesante, tendré que mirar el documento al que hace referencia con más cuidado ...

Respuestas (3)

Alice prepara un campo electromagnético en un estado con un gran número de fotones norte ^ | norte = norte | norte dónde norte ^ es el operador numérico. Alice se alienta con respecto a Bob. En el marco de referencia de Bob, el campo está en el estado tu ^ ( Λ ) | norte . La pregunta es si una medida del número de fotones para el estado de Bob da la respuesta precisa. norte . En otras palabras, ¿es cierto que norte ^ tu ^ ( Λ ) | norte = norte tu ^ ( Λ ) | norte ? Bob obtendrá el resultado nítido. norte si el operador impulso conmuta con el operador numérico. Sólo tenemos que demostrar que el conmutador [ tu ^ ( Λ ) , norte ^ ] = 0 .

El operador numérico para fotones de helicidad. λ es,

norte λ ^ = d 3 pags 2 ω η ^ pags λ η ^ pags λ
dónde η ^ pags λ , η ^ pags λ son operadores de emisión y absorción respectivamente para un fotón de momento pags y helicidad λ (la notación para los operadores de emisión y absorción es de la monografía de Dirac "Lectures on Quantum Field Theory"). También tenemos ω = pags 0 en la medida invariante de Lorentz.

Los estados de un solo fotón se transforman como,

tu ^ ( Λ ) | pags , λ = mi i θ ( pags , Λ ) | Λ pags , λ
dónde θ ( pags , Λ ) es el ángulo de Wigner. Creando un estado de una sola partícula a partir del vacío. | S por | pags , λ = η ^ pags λ | S implica que los operadores de emisión transforman estados similares,
tu ^ ( Λ ) η ^ pags λ = mi i θ ( pags , Λ ) η ^ Λ pags λ   .
Tomando el conjugado hermitiano, usando la unitaridad y reemplazando Λ por Λ 1 ,
η ^ pags λ tu ^ ( Λ ) = mi i θ ( pags , Λ ) η ^ Λ pags λ η ^ pags λ tu ^ ( Λ 1 ) = mi i θ ( pags , Λ ) η ^ Λ pags λ η ^ pags λ tu ^ ( Λ ) = mi i θ ( pags , Λ 1 ) η ^ Λ 1 pags λ   .
Ahora evalúe el conmutador,
[ tu ^ ( Λ ) , norte ^ λ ] = d 3 pags 2 ω tu ^ ( Λ ) η ^ pags λ η ^ pags λ d 3 pags 2 ω η ^ pags λ η ^ pags λ tu ^ ( Λ ) = d 3 pags 2 ω mi i θ ( pags , Λ ) η ^ Λ pags λ η ^ pags λ d 3 pags 2 ω η ^ pags λ mi i θ ( pags , Λ 1 ) η ^ Λ 1 pags λ   .
Realiza un cambio de variable en la segunda integral, pags = Λ 1 pags .
[ tu ^ ( Λ ) , norte ^ λ ] = d 3 pags 2 ω mi i θ ( pags , Λ ) η ^ Λ pags λ η ^ pags λ d 3 pags 2 ω η ^ Λ pags λ mi i θ ( Λ pags , Λ 1 ) η ^ pags λ
El ángulo de Wigner θ ( pags , Λ ) corresponde a una matriz de rotación R ( pags , Λ ) = H Λ pags 1 Λ H pags dónde H pags es el impulso estándar. Ahora,
R ( Λ pags , Λ 1 ) = H Λ 1 Λ pags 1 Λ 1 H Λ pags = H pags 1 Λ 1 H Λ pags = ( H Λ pags 1 Λ H pags ) 1 = ( R ( pags , Λ ) ) 1
de modo que el ángulo de Wigner θ ( Λ pags , Λ 1 ) es θ ( pags , Λ ) . Al poner este resultado en la última integral, el conmutador desaparece. [ tu ^ ( Λ ) , norte ^ λ ] = 0 y entonces el campo electromagnético de Bob también tiene el mismo número agudo norte de fotones como el campo de Alice.

Editar: Explicación de por qué aparece la medida invariante en el operador numérico

El método de representaciones inducidas, que se utiliza para obtener la respuesta de los estados de una sola partícula a un impulso de Lorentz (segunda ecuación en el texto principal), es más simple si se elige una medida invariante de Lorentz para que la resolución de la unidad para los estados de una sola partícula es,

λ = ± 1 d 3 pags 2 ω | pags , λ pags , λ | = 1   .
Esta elección implica que el conmutador de los operadores de emisión y absorción es,
[ η ^ pags λ , η ^ pags λ ] = pags , λ | pags , λ = 2 ω d λ , λ d 3 ( pags pags )   .
A su vez, esto implica que el hamiltoniano de orden normal para el campo electromagnético libre es,
H ^ = 1 2 d 3 pags ( η ^ pags λ = 1 η ^ pags λ = 1 + η ^ pags λ = + 1 η ^ pags λ = + 1 )   .
Ahora crea norte fotones del vacío con un estado,
| Ψ = ( η ^ pags λ ) norte | S
y exigir que el operador de números norte ^ λ mide el resultado nítido norte sobre este estado. Esto implica que la medida invariante de Lorentz debe usarse en la definición del operador numérico (primera ecuación en el texto principal). Entonces, uno ve que no hay suposiciones aquí, solo una elección de la medida invariante (en lugar de una medida cuasi-invariante) para hacer que el método de representaciones inducidas utilizado para obtener los irreps del grupo de Poincaré para partículas sin masa sea lo más simple posible. .

La gran suposición aquí es cómo los estados del espacio de Fock y los operadores de creación se transforman frente al grupo de Lorentz (es decir,
tu ^ ( Λ ) | pags , λ = mi i θ ( pags , Λ ) | Λ pags , λ
). Una vez que se hacen esas suposiciones, el resultado es sencillo. ¿Son válidas esas suposiciones? Dado que la ecuación de onda es covariante de Lorentz y sus soluciones traen operadores de creación, diría que esas suposiciones son válidas en el caso libre y consistentes. En el caso de interacción es menos claro. En un marco acelerado, el efecto Unruh muestra claramente que esto no funciona
En cuanto a cualquier conversación sobre el componente cero del impulso 4, esto es irrelevante. Nadie esperaría que el hamiltoniano fuera covariante de Lorentz. Claramente habría un desplazamiento rojo/azul, pero eso no dice nada sobre el número de partículas que se desplazan hacia el rojo/azul, lo que debería ser invariante de Lorentz si hacemos las suposiciones anteriores sobre cómo se transforma el espacio de Fock.
@Saleh Hamdan: La respuesta de los estados de una sola partícula a un impulso de Lorentz (segunda ecuación en mi respuesta) no es una suposición. Es una consecuencia de encontrar las representaciones irreducibles del grupo de Poincaré usando el método de representaciones inducidas (ver "Representaciones Inducidas de Grupos y Mecánica Cuántica" por George W. Mackey, WA Benjamin, 1968) para el caso sin masa. Estas representaciones solo se derivan para partículas libres y el grupo de Poincaré solo funciona entre marcos de referencia que no aceleran, como señala.
La "Explicación de por qué aparece la medida invariante en el operador numérico" acabó con cualquier duda. Gracias
¿Cómo se concilia la respuesta con la respuesta aceptada de physics.stackexchange.com/questions/21830/… ?

La respuesta de un experimentador.

Hay innumerables experimentos que miden dos eventos gamma. La invariancia de Lorentz es una suposición básica para todas las interacciones medidas. Cada interacción está en un marco de Lorenz diferente según las energías y los momentos involucrados. Cuando hacemos las distribuciones de secciones transversales y ángulos, dependemos de esta invariancia del número de partículas en la interacción bajo observación. Como el modelo estándar se las arregla para ajustar todo esto con una muy buena aproximación, esta suposición se mantiene.

Ahora cada fotón individual proviene de una interacción invariante de Lorenz por construcción de interacciones electromagnéticas, aunque nada lo registra, por lo que los números deben permanecer constantes.

Para que los números cambien si el marco de Lorenz cambia para un conjunto de fotones ya creados, significa que el marco de Lorenz impuesto interactúa de alguna manera con los fotones bajo observación. Si se intercambia energía, pueden aparecer más fotones, lo que parecerá una falta de conservación de los números, pero no debe considerarse como tal.

Usted habla de experimentos con eventos gamma, pero es posible que no sean demasiado sensibles, por ejemplo, a algunos fotones de microondas que aparecen en otros cuadros.
@Ruslan Si la producción de fotones, el número de fotones, no fuera invariante de Lorenz, las secciones transversales calculadas que provienen de la hipótesis de la invariancia de Lorenz no encajarían. Los datos para el decaimiento de pi0, por ejemplo, cubren una enorme gama de energías del pi0, cada uno es un marco de lorenz diferente con respecto a los otros decaimientos. Mi punto es que para generar un fotón necesitas una interacción, no se generan a partir del vacío.

No estoy seguro de quién está eliminando mis comentarios, argumentos y críticas que cuestionan la derivación del Sr. Blake. El hecho de que no tengamos suficiente razonamiento para argumentar en contra de una idea nunca debería significar borrar esa voz. Hasta donde yo sé, la ciencia no está destinada a ser monopolizada, ya que lo mismo ocurre con la riqueza.

Al contar el número de fotones en un sistema, estamos limitados por la detección, que a su vez está limitada por el Principio de Incertidumbre de Heisenberg, algo que nunca se puede superar en la Naturaleza. En teoría, un fotón puede tener cualquier energía distinta de cero en un espectro infinitamente amplio. Por lo tanto, existe una probabilidad distinta de cero de que algún fotón tenga una energía particular. No olvidemos que los fotones son solo un ingrediente de las partículas fundamentales en la Naturaleza. Esto significa que pueden interactuar con al menos cualquier otra partícula fundamental cargada del Modelo Estándar, incluidas sus propias interacciones. (Los canales de interacción aumentarían exponencialmente a medida que vayamos más allá de las teorías del modelo estándar, como las dimensiones extra y la supersimetría). Dicho todo esto, está perfectamente bien que los fotones se pierdan y/o se creen (a través de sus interacciones) y nunca se compensen en un volumen finito dado de nuestro Universo (que se llamará sistema) durante un período de tiempo finito (dentro de la edad finita del Universo). Por lo tanto, al ser solo un subconjunto de todas las partículas fundamentales y tener un tiempo de vida infinitamente largo durante el cual tienen muchos canales de interacción, su número nunca se conservará en ningún volumen finito de nuestro Universo. Sin embargo, sería más desafiante generalizar la pregunta a "si se conserva el número total de todas las partículas fundamentales dentro del Universo Visible". Sin embargo, para responder a esto, necesitamos conocer la Teoría del Todo en la que se conocen TODAS las partículas fundamentales, incluidos sus tiempos de vida y masas y canales de interacción junto con el conocimiento de la Materia Oscura y la Energía Oscura (que representan el 96% del presupuesto de materia y energía del Universo que son invisibles a partir de Este Dia). Entonces, estoy seguro de que la respuesta a su pregunta es: "No. El número de fotones en un volumen finito conocido como sistema nunca se conservará". Y me gustaría dejar el más desafiante para las futuras generaciones de físicos.

No voté negativo. Pero los fotones no están cargados, así que no digas "cualquier otra partícula cargada". Y conservado es diferente del marco invariante, por lo que su respuesta ha entendido mal la pregunta. Y el universo ni siquiera tiene que estar en un estado de número de partículas definido e incluso si lo fuera momentáneamente, sabemos que podemos cambiar el número de partículas, hacemos experimentos y observaciones que involucran esto todo el tiempo. Entonces no se conserva, y la pregunta era sobre la invariancia del marco de todos modos, que es totalmente diferente.
No estoy seguro de quién está eliminando mis comentarios, argumentos y críticas que cuestionan la derivación del Sr. Blake. El hecho de que no tengamos suficiente razonamiento para argumentar en contra de una idea nunca debería significar borrar esa voz. Hasta donde yo sé, la ciencia no está destinada a ser monopolizada, ya que lo mismo ocurre con la riqueza.
No he eliminado sus comentarios a Stephen Blake, pero este es el segundo comentario suyo que veo que parece haber sido colocado como respuesta a mi comentario. ¿Estás seguro de que estás comentando la respuesta de Blake? Y los comentarios no son para discusiones de todos modos, están enfocados en mejorar las respuestas. También son de segunda clase en el sentido de que no se espera que sobrevivan. Puedes votar a favor o en contra y eso se mantiene. Se supone que un comentario fomenta la modificación de una respuesta y eso es realmente para lo que sirve. Pero no se puede forzar un cambio en una respuesta.
Hola Timaeus, gracias por avisarme. Sí, estoy seguro de que estoy comentando la respuesta de Blake. Realmente no sé qué está pasando aquí. Estaba esperando que me respondiera, pero parece que mi pregunta se borra constantemente. Sin embargo, voy a ser más paciente hasta que se resuelva el problema. Sinceramente,
Usted es libre de elaborar una mejor respuesta usted mismo.